Carl used 10 2/9 of an inch of string to tie a parcel and another 5 1/4 of an inch of string to
tie a box, how much string is left if he started with 20 inches?

Answers

Answer 1

Answer: 4 19/36 inches

Step-by-step explanation:
To add the two measurements together, you need to find a common denominator. In this instance, it will be 36.

2 * 4 = 8, 9 * 4 = 36, which means our first measurement turns into 10 8/36

1 * 9 = 9, 4 * 9 = 36, which means our second measurement turns into 5 9/36

Adding these together we get 15 17/36.

Subtract this from 20.

The amount of string that is left is 4 19/36 inches.

Hope this helped!


Related Questions

Are these ratios proportional ?
1/4 and 5/20 yes or no

Please help! I’m so stressed I’ll give 25 points I just need this

Are these ratios proportional ? 1/4 and 5/20 yes or noPlease help! Im so stressed Ill give 25 points

Answers

Answer:

yes

Step-by-step explanation:

if you simplify 5/20 it is 1/4

Answer:

yes

Step-by-step explanation:

Estimate ΔyΔy using differentials.
y=cos(5x),=/30,x=0.055
(Give your answer to three decimal places.)

Answers

The estimated change in yy using differentials is -0.00679. This means that if xx is increased by 0.005, then yy is estimated to decrease by 0.00679. The differential of yy is dy=-5sin(5x)dxdy=−5sin⁡(5x)dx. We are given that y=cos(5x)=π/30y=cos⁡(5x)=π/30 and x=0.055x=0.055.

We want to estimate ΔyΔy, which is the change in yy when xx is increased by 0.005. We can use the differential to estimate ΔyΔy as follows:

Δy≈dy≈dy=-5sin(5x)dx

Plugging in the values of y, x, and dxdx, we get:

Δy≈-5sin(5(0.055))(0.005)≈-0.00679

Therefore, the estimated change in yy using differentials is -0.00679.

To learn more about differential click here : brainly.com/question/31383100

#SPJ11

help me please !!! correct answer gets brainliest

help me please !!! correct answer gets brainliest

Answers

Answer: 1/10 is literally only one out of 10. the fraction to a decimal is 0.1 so, 3 x 0.1 is 0.3 and 3 x 10 is 30.

0.3 is less than 30.

Step-by-step explanation:

I NEEDHELP ASAP IN ON TIMER !!!!
Which polynomial is in standard form?
A. 9+2x-8x4+16x5
B. 12x5-6x2-9x+12
C. 13x5+11x-6x2+5
D. 7x7+14x9-17x+25

Answers

I think it’s is A if not c

Write the decimal equivalent for each rational number. Use a bar over any repeating digits 3/11

Answers

Answer:

see below

Step-by-step explanation:

1/11=0.0909....

3/11=0.090909...*3 or 0.27272727...

Answer the question in photo [ Brainliest + easy points ]

Answer the question in photo [ Brainliest + easy points ]

Answers

Answer:

for This you have to find the value of x.

Step-by-step explanation:

just add both of them the equation formed will be

22x - 26 = 19x -17

so you can calculate x

which is x = 3

so put the value in 14x -15

so TU will be 27

if you have any doubt you can comment it.

thankyou .

Questian 14 (1 point) A solid metal prism has square base with sides of 4 inches, and height of 6 inches: A hole in the shape of a cylinder; with radius of inch, is drilled through the entire length of the rectangular prism. What is the approximate volume of the remaining solid, in cubic inches? 19 cubic inches 77 cubic inches 96 cubic inches 93 cubic inches 6 In

Answers

Rounding to the nearest whole number, we get an approximate volume of 91.6  cubic inches. Therefore, the answer is not one of the provided options.

As given, a solid metal prism has a square base with sides of 4 inches and a height of 6 inches. A hole in the shape of a cylinder with a radius of inch is drilled through the entire length of the rectangular prism.To calculate the approximate volume of the remaining solid, we need to find the volume of the rectangular prism and subtract the volume of the cylinder from it.

To find the volume of the rectangular prism, we can use the formula

Volume of a rectangular prism = Base area × Height

Since the base of the rectangular prism is square, we can use the formula for the area of a square to find the base area.

Base area = side²= 4² = 16 square inches

Now, we can calculate the volume of the rectangular prism as

Volume of rectangular prism = Base area × Height= 16 × 6= 96 cubic inches

Next, we need to find the volume of the cylinder. The formula for the volume of a cylinder is

Volume of a cylinder = π × r² × h

where r is the radius and h is the height of the cylinder. The height of the cylinder is the same as the height of the rectangular prism, which is 6 inches. The radius of the cylinder is given as inch, which is 0.5 inches in decimal form. So, we can calculate the volume of the cylinder as

Volume of cylinder = π × (0.5)² × 6= π × 0.25 × 6= 1.5π cubic inches

Now, we can find the volume of the remaining solid by subtracting the volume of the cylinder from the volume of the rectangular prism.Volume of remaining solid = Volume of rectangular prism - Volume of cylinder= 96 - 1.5π ≈ 91.6 cubic inches (approx)

for more questions on volume

https://brainly.com/question/12004994

#SPJ11

E QUESTION ONE
Solve the proportion—Show all work.
5
x 1 W
=
65
X Х
XX

E QUESTION ONESolve the proportionShow all work.5x 1 W=65X XX

Answers

Answer:

x=39

Step-by-step explanation:

3/x  = 5/65

5x=195

x=39

Answer:

39

Step-by-step explanation:

The ratio of males to females was 4:5. If there are 180 people at the fair how many males
are there?

Answers

9514 1404 393

Answer:

  80

Step-by-step explanation:

Let m represent the number of males. Then you have ...

  m/(180 -m) = 4/5

  5m = 4(180 -m) . . . . cross multiply

  9m = 720 . . . . . . . . add 4m, simplify

  m = 80 . . . . . . . . . . divide by 9

The number of males at the fair was 80.

For the solution to the inequality in the previous problem, list all integers from -10 through 10 that will make the
inequality a true statement when substituted for "n"? (The integers for -10 through 10 are shown below.)
{-10,-9,-8, -7,-6, -5, -4,-3,-2,-1,0, 1, 2, 3, 4, 5, 6, 7, 8, 9, 10}
HELP PLEASE HURRY
ILL GIVE 30 POINTS AND MOST BRAINLIEST

For the solution to the inequality in the previous problem, list all integers from -10 through 10 that

Answers

Answer: -10, -7, -5, -2, -1, 3, 4, 8, 9

Step-by-step explanation:

-6(-3x-3)=6(1+3x) solve

Answers

Answer:

x=-0.6

Step-by-step explanation:

-3x-3=-1

-3x=-1+3

-3x=2

x=-2/3

What is the volume of the following rectangular prism?
8/3units
21/2units2
Volume =___ units3​

What is the volume of the following rectangular prism?8/3units21/2units2Volume =___ units3

Answers

Answer:

73.5

Step-by-step explanation:

Step 1: 21/2 divided by 2 = 21/4

Step 2: 21/4 times 21/4 times 8/3 =

Answer: 73.5

The green shaded box is there to show that that is the base are to find the two sides that were multiplied to get that do the first step. Then do your basic volume formula: length times width times height.

What is the slope of the line passing through the points (0,4) and (-8, -1)?

A. -8/5

B. -5/8

C. 5/8

D. 8/5

Answers

Answer:

5/8

Step-by-step explanation:

\( slope \: = \frac{( - 1) - 4}{( - 8) - 0} = \frac{5}{8} \)

Answer: B. -5/8

Step-by-step explanation:

Consider these numbers, ordered from least to greatest. Negative 1.8, negative 1 and one-fourth, negative 0.2, blank, one-half, 90 percent A number line going from negative 2 to positive 1. Which values could correctly fill in the blank? Select all that apply.

Answers

Answer:

10%

-0.1

1/4

Step-by-step explanation:

Complete question

Consider these numbers, ordered from least to greatest. Negative 1.8, negative 1 and one-fourth, negative 0.2, blank, one-half, 90 percent A number line going from negative 2 to positive 1. Which values could correctly fill in the blank? Select all that apply. 10% 1 -0.1 -1 One-fourth

step-by-step explanation

On the number line, numbers increase from left to right.

The blank is between -0.2 and 1/2.

Only numbers greater than -0.2 and less than 1/2 can fill the blank.

Range = -0.2<x<1/2

From the values given,

1) 10% = 10/100 = 1/10 = 0.1

-0.2<0.1<1/2

2) -0.2<1

But, 1>1/2

3) -0.2<-0.1<1/2

4) -0.2>-1

5) -0.2<1/4<1/2

Therefore, only 10%, -0.1, and 1/4 can fill in the blank correctly because they satisfy the range -0.2<x<1/2

Consider these numbers, ordered from least to greatest. Negative 1.8, negative 1 and one-fourth, negative

Answer:

A

C

E

THE ANSWER BOIIIIIIIIIIIIIIIIIIIIIII!

Step-by-step explanation:

what is 6x-18/x^2 * X^2+5x+6/x^2-9 ANSWER FAST!!!!!

Answers

9514 1404 393

Answer:

  (6x+12)/x^2

Step-by-step explanation:

The product can be simplified by cancelling common factors from numerator and denominator.

  \(\dfrac{6x-18}{x^2}\times\dfrac{x^2+5x+6}{x^2-9}=\dfrac{6(x-3)}{x^2}\times\dfrac{(x+2)(x+3)}{(x-3)(x+3)}\\\\=\dfrac{6(x+2)}{x^2}=\boxed{\dfrac{6x+12}{x^2}}\)

2-simplifica

1)x²-5x-16

x+2=

2)6an²-3b²n²

b4-4ab²+4a²=

3)4x²-4xy+y²

5y-10x

4)n+1-n³-n²

n³-n-2n²+2=

5)17x³y4z6

34x7y8z10=

6)12a²b³

60a³b5x6=

Answers

1.  x² - 5x - 16 can be written as (x - 8)(x + 2).

2. 6an² - 3b²n² = n²(6a - 3b²).

3. This expression represents a perfect square trinomial, which can be factored as (2x - y)².

4. Combining like terms, we get -n³ - n² + n + 1 = -(n³ + n² - n - 1).

5. 17x³y⁴z⁶ = (x²y²z³)².

6. 12a²b³ = (2a)(6b³) = 12a6b³ = 12a⁷b³x⁶.

Let's simplify the given expressions:

Simplifying x² - 5x - 16:

To factorize this quadratic expression, we look for two numbers whose product is equal to -16 and whose sum is equal to -5. The numbers are -8 and 2.

Therefore, x² - 5x - 16 can be written as (x - 8)(x + 2).

Simplifying 6an² - 3b²n²:

To simplify this expression, we can factor out the common term n² from both terms:

6an² - 3b²n² = n²(6a - 3b²).

Simplifying 4x² - 4xy + y²:

This expression represents a perfect square trinomial, which can be factored as (2x - y)².

Simplifying n + 1 - n³ - n²:

Rearranging the terms, we have -n³ - n² + n + 1.

Combining like terms, we get -n³ - n² + n + 1 = -(n³ + n² - n - 1).

Simplifying 17x³y⁴z⁶:

To simplify this expression, we can divide each exponent by 2 to simplify it as much as possible:

17x³y⁴z⁶ = (x²y²z³)².

Simplifying 12a²b³:

To simplify this expression, we can multiply the exponents of a and b with the given expression:

12a²b³ = (2a)(6b³) = 12a6b³ = 12a⁷b³x⁶.

Learn more about  expression from

https://brainly.com/question/723406

#SPJ11

Sabia and arlan bougth 45 book for r 30 each old all ok them at the rote of 24 find the different the money pent and return

Answers

If Sabia and Arlan bought 45 books for rs 30, then at the rate of rs 24, they can buy 34 books.

What is an arithmetic operation?

It is defined as the operation in which we do the addition of numbers, subtraction, multiplication, and division. It has basic four operators that are +, -, ×, and ÷.

It is given that, Sabia and Arlan bought 45 books for rs 30, then at the rate of rs 24, how many books they can buy,

Rs 30 = 45 book

Rs 30 = 45

Rs 1 = 45 / 30

Rs 24 = 3/2 × 24

Rs 24 = 34 books

Thus, if Sabia and Arlan bought 45 books for rs 30, then at the rate of rs 24, they can buy 34 books.

Learn more about the arithmetic operation here:

brainly.com/question/20595275

#SPJ1

Use the product rule to find the derivative of the function. Select the correct answer below and fill in the answer box(es) to complete your choice. The derivative is (x-50+(x+1)。 A. B. The derivative is (x-5)(x + 1) C. The derivative is (x-5) O D. The derivative is (x(1x+1) OE. The derivative is (x-5 1x+1)+

Answers

The correct answer is B. The derivative is 2x - 49.

To find the derivative of the function (x - 50 + (x + 1)), we can apply the product rule, which states that the derivative of the product of two functions is the first function times the derivative of the second function, plus the second function times the derivative of the first function.

Let's break down the given function into its components:

f(x) = (x - 50) + (x + 1)

To differentiate this function, we can consider each term separately:

Term 1: (x - 50)

Term 2: (x + 1)

Now, applying the product rule, we differentiate each term and combine the results:

Derivative of Term 1 = 1  (since the derivative of x is 1)

Derivative of Term 2 = 1  (since the derivative of x is 1)

Using the product rule, the derivative of the function is:

f'(x) = (x - 50) * 1 + (x + 1) * 1

f'(x) = x - 50 + x + 1

f'(x) = 2x - 49

Therefore, the correct answer is B. The derivative is 2x - 49.

To learn more about derivative from the given link

https://brainly.com/question/31136431

#SPJ4

Solve the triangle. A=35∘,B=35∘,c=6 C= (Do not round until the final answer. Then round to the nearest degree as needed.) a≈ (Do not round until the final answer. Then round to the nearest tenth as needed.) b≈ (Do not round until the final answer. Then round to the nearest tenth as needed.)

Answers

a ≈ (6 * sin(35°)) / sin(110°), b ≈ (6 * sin(35°)) / sin(110°). The Law of Sines states that the ratio of the length of a side to the sine of its opposite angle is constant for all sides in a triangle.

**Answer:** In triangle ABC, where A = 35°, B = 35°, and c = 6, we need to find the values of C, a, and b.

To find the missing angle, we can use the fact that the sum of all angles in a triangle is always 180°. Therefore, C = 180° - A - B = 180° - 35° - 35° = 110°.

Next, we can use the Law of Sines to find the lengths of sides a and b. The Law of Sines states that the ratio of the length of a side to the sine of its opposite angle is constant for all sides in a triangle. In this case, we can write:

a/sin(A) = c/sin(C)   (1)

b/sin(B) = c/sin(C)   (2)

Substituting the known values, we have:

a/sin(35°) = 6/sin(110°)   (3)

b/sin(35°) = 6/sin(110°)   (4)

Solving equations (3) and (4) simultaneously will give us the values of a and b.

By cross-multiplying equation (3), we get:

a * sin(110°) = 6 * sin(35°)

a ≈ (6 * sin(35°)) / sin(110°)

Using a calculator, we can evaluate this expression to find the approximate value of a.

Similarly, by cross-multiplying equation (4), we get:

b * sin(110°) = 6 * sin(35°)

b ≈ (6 * sin(35°)) / sin(110°)

Again, using a calculator, we can evaluate this expression to find the approximate value of b.

After rounding to the nearest tenth, we will have the final approximated values for a and b.

Learn more about triangle here

https://brainly.com/question/17335144

#SPJ11

. a simple undirected graph has 10 edges. 2 of the vertices are of degree 4, and the rest of the vertices are of degree 3. how many vertices are in this graph?

Answers

There are 4 vertices with degree 3, and 2 vertices with degree 4. So, there are a total of 6 vertices in this graph.

In a simple undirected graph with 10 edges, 2 vertices have a degree of 4 and the rest have a degree of 3. To determine the number of vertices in this graph, use the formula:

Sum of degrees = 2 * number of edges

Let x be the number of vertices with degree 3. Then:

(2 * 4) + (3 * x) = 2 * 10

8 + 3x = 20

3x = 12

x = 4

We also know that two of the vertices have a degree of 4, which means that together they contribute 8 to the sum of all vertex degrees. This leaves us with 20 - 8 = 12 degrees left to distribute among the other vertices. Since the remaining vertices all have a degree of 3, we can set up the equation 3x = 12, where x is the number of remaining vertices. Solving for x, we get x = 4. Therefore, the graph has a total of 6 vertices - two with a degree of 4 and four with a degree of 3.

Know  more about vertices here:

https://brainly.com/question/27029296

#SPJ11

An airplane is flying from New York City to Los Angeles. The distance it travels in miles, d, is related to the time in seconds, t, by the equation d=0.15t.
How fast is it flying?
How long will it take to go 12.75 miles?

Answers

Answer:

Q: How long will it take to go 12.75 miles?

A: 85 seconds

Step-by-step explanation:

d = 0.15t

12.75 = 0.15t

0.15/ 12.75 = 85

Sorry,I don't know how to answer the first question(How fast is it flying?)

Hoped this helped though in some way

if f =28 when a = 7 find f when a is 10 (direct proportion) ​

Answers

Answer:

f=40

Step-by-step explanation:

f=28, a=7

\(f \infty a \\ f = ka \\28 = k(7) \\ 28 = 7k \\ \frac{28}{7} = \frac{7k}{7} \\ 4 = k \\ k = 4 \\ f = 4a \\ a = 10 \\ f = 4(10) \\ f = 40\)

Hope that this is helpful.

Have a nice day.

F=40
Don’t really know how to explain it tho

May someone please answer this question fast? :3

May someone please answer this question fast? :3

Answers

Answer:

3*3*3*3*3

Step-by-step explanation:

exponents are how many times a number is multipleid by iteself

Answer:

3*3*3*3*3 = 243

farmer ed has 9000 meters of fencing, and wants to enclose a rectangular plot that borders on a river. if farmer ed does not fence the side along the river, find the length and width of the plot that will maximize the area. what is the largest area that can be enclosed?

Answers

The length and width of the plot that will maximize the area is 60 meters by 120 meters. The largest area that can be enclosed is 7200 square meters.

To maximize the area of the rectangular plot, Farmer Ed must use 9000 meters of fencing to enclose three sides of the plot. The fourth side, which borders the river, does not need to be fenced. To find the length and width of the plot that will maximize the area, the 9000 meters of fencing must be divided into two sides of equal length, with the remaining fencing used for the third side. This results in a length of 120 meters and a width of 60 meters, which maximizes the area of the plot. The largest area that can be enclosed is 7200 square meters.

learn more about area here

https://brainly.com/question/27683633

#SPJ4

in theyx, y-plane above, the circle has center (h, k)left parenthesis, h, comma, k, right parenthesis and radius 10. what is the value of k ?

Answers

We can say that the value of k will be equal to the y-coordinate of the center of the circle (since k represents the y-coordinate of the center).

To answer this question, we need to use the equation of a circle:

(x - h)^2 + (y - k)^2 = r^2

Where h and k represent the coordinates of the center of the circle, and r represents the radius. In this case, we are given that the circle has center (h, k) and radius 10. So we can write:

(x - h)^2 + (y - k)^2 = 10^2

We are asked to find the value of k. To do this, we need to look at the equation of the circle and notice that the y-coordinate is paired with k. This means that we can isolate k by rearranging the equation as follows:

(y - k)^2 = 10^2 - (x - h)^2

y - k = ±√(10^2 - (x - h)^2)

k = y ±√(10^2 - (x - h)^2)

Since we don't have any information about the x-coordinate, we can't solve for a specific value of k. However, we can say that the value of k will be equal to the y-coordinate of the center of the circle (since k represents the y-coordinate of the center). Therefore, the answer is:

k = y

To know more about Coordinate  visit :

https://brainly.com/question/22261383

#SPJ11

Simplify the following fraction WITHOUT USING A CALCULATOR tan 45°.sin 30-cot45°​

Answers

Answer:

tan 45°.sin 30° - cot45°​ = −(1÷2) = −0.5

Step-by-step explanation:

45° Corresponds to (π÷4) which is a remarkable measure

We are all supposed to learn by heart the values of it’s ‘sin’ , ‘cos’ , ‘tan’ and ‘cot’ .

the same apply to 30° (corresponds to (π÷6))

……………………………………………………………

So without a calculator:

tan 45° = 1

cot 45° = 1

sin 30°​ = 1/2

therefore ,

tan 45°.sin 30° - cot45°​ = 1 × (1÷2) − 1 = (1÷2) − 1 = −(1÷2) = −0.5

!!!Need Help ASAP

R=42−0.7t


Quinn returned home one summer's day to find it extremely hot. He turned the air conditioner on, and the room's temperature began decreasing at a constant rate. The equation shown above gives the room's temperature, R, in degrees Celsius, t minutes after Quinn turned on the air conditioner. What was the room's temperature, in degrees Celsius, when Quinn returned home?

Answers

The room's temperature when Quinn returned home is the y-intercept of the equation, which is: 40 degrees Celsius.

What is the Y-intercept or Initial Value of a Linear Equation?

The equation of a linear relationship between two variables, x and y, can be expressed in slope-intercept form as y = mx + b, where we have the initial value or the y-intercept as "b", and the rate of change or unit rate as "m".

Alternatively, it can be rewritten as y = b + mx.

R = 42 - 0.7t represents the equation for the Quinn's room's temperature, R, in degrees Celsius, and t in minutes after Quinn turned on the air conditioner.

Interpreting the equation, we can deduce the following:

The initial value or the y-intercept (b) was the room's temperature when Quinn returned home.

The unit rate or rate of change (m) = -0.7 degrees per minute.

Thus, Quinn's room's temperature when she returned was 40 degrees Celsius.

Learn more about y-intercept of equation on:

https://brainly.com/question/15602982

#SPJ1

3x - 4 = 17 solve for x

Answers

Answer:

x=7

Step-by-step explanation:

3(7) - 4=17

21-4=17

x=7!!!!!!!!!!!!!! !!!!!!!!!!!!!

A rectangle has an area of 72 square units. The width of the rectangle is 9 units. The length of the rectangle is 2x + 4.
What is the rectangle's length?
A. 6
B. 7
C. 8
D. 9

Answers

Answer:

8

Step-by-step explanation:

If you plug in the answers it will give you 9x6= 54 which is wrong and 9x7=63 and 9x9=81 but 9x8=72.

Answer:

C   length = 8

Step-by-step explanation:

Area of Rectangle = lw

A = lw    Substitute l = (2x + 4) ; w = 9

72 = (2x + 4)9

72 = 18x + 36

36 =        -36

36 = 18x

36/18 = x

 2 = x

 Substitute into l = 2x + 4 ; l = 2(2) + 4; l = 4 + 4 ; l = 8

1. You want to plant a flower garden in your yard so that you can make a beautiful bouquet to put on the alter at church for Easter Sunday services. There are two types of flowers that you are going to be planting. You will be planting some tulips and daffodils. At the store tulips come in packs of 6 and daffodils come in packs of 9.
a.What is the least amount of packs of daffodils and tulips you would need to buy to have the same amount of each?

b.How many should you have of each kind of flower?


2.Each pack of tulips costs $4.75 and each pack of daffodils cost $3.50. From the previous problem, you know how many packs of each you will need. The sales tax is 7.5%.
a.How much tax will you be paying for all the flowers, to the nearest cent?



b.How much total will you be paying for all flowers, to the nearest cent?


3.You remember that you have a discount coupon for $2.75 off your total purchase before tax.
a. How much did the flowers cost with the discount including tax (Round to the nearest penny)?

b.How much money did you save with the coupon including tax?


4.When you get home to plant the flowers, you have 3 different flowerbeds in which you plan on planting flowers. You have two flowerbeds where you want the same ratio for tulips to daffodils. In one of these flowerbeds you have 4 tulips and 6 daffodils. The other flowerbed you have 3 daffodils.


a. How many tulips should there be in the second flowerbed to have the same ratio of tulips and daffodils as the first flowerbed?


b.How many total daffodils are used in these two flowerbeds?


c.How many total tulips are to be used in these two flowerbeds?


d.How many of each do we have left to use in the remaining flowerbed?


5.The flowers are supposed to get 1.5 cups of water per plant. You have a 3-quart water pail.

How many flowers can you water before having to go back to fill up your pail?


6.After a few days 1/3 of all your tulips and 1/2 of all your daffodils died.

How many tulips and how many daffodils do you need to purchase of each kind of plant to replace the flowers that died?



7.Next year you want to have 7 more than double the amount of plants you had this year.

How many total plants should you have for next year?

Answers

The solution to each total amount is as follows

1a.   3 packs of tulips and  2 packs of daffodils.  1b. 18  tulips and 18 daffodils.

2a.  The tax  $1.60    2b. The total cost $22.85.

3. total cost with discount and tax is  $19.89.  3b. Amount saved is $2.95.

4. 2 tulips.   4b. 9 daffodils    4c.  6 tulips.  4d. 12 tulips  and 9 daffodils.

5. 8 flowers before having to refill the pail.

6.  you need to purchase 6 tulips and 9 daffodils.

7. you should aim to have 79 plants next year.

How did we solve for each amount of tulips and daffodils?

1. least common multiple of 6 and 9 is 3. 6/3 = 2 and 9/3 = 3.

Cross multiply ⇒ 9 x 2 = 18  and 6 x 3 = 18

2. The cost of tulip and daffodil packs are

3×$4.75 = $14.25

2×$3.50 = $7.

The total cost before tax⇒ $14.25 + $7 = $21.25.

The tax⇒ $21.25 × 7.5/100 = $1.60 (rounded to the nearest cent).

The total cost including tax is $21.25 + $1.59 = $22.85.

3. Before tax ⇒ $21.25.

After applying a $2.75 discount, ⇒ $21.25 - $2.75 = $18.50.

The tax on this discounted amount is $18.50 × 7.5/100 = $1.39  

Total cost with discount and tax is $18.50 + $1.39 = $19.89. (rounded to the nearest penny).

The total amount saved $22.85 - $19.89 = $2.95

4. ratio of tulips to daffodils is 4:6. simplify ⇒  2:3.  for every 3 daffodils, 2 tulips

The total number of daffodils 6 + 3 = 9

The total number of tulips 4 + 2 = 6

18 - 6 = 12 tulips and 18 - 9 = 9 daffodils are left for the remaining flowerbed

5. A 3-quart⇒ 12 cups ⇒ 1 quart = 4 cups.

∴ 12/1.5 = 8 flowers before refill.

6. 1/3 of the tulips died, ⇒ 1/3 ×18 = 6 tulips.

1/2 of the daffodils died, ⇒1/2 × 18 = 9 daffodils.

7.  18 tulips and 18 daffodils for a total of 36 plants.

36×2 = 72 + 7 = 79

Find more exercise on finding total amounts;

https://brainly.com/question/28000147

#SPJ1

Other Questions
Siambanopolis CompanyPresented below are selected transactions from Siambanopolis Company for 2015. Amortization is calculated on a straight-line basis. You will have to calculate accumulated amortization. Journalize each transaction.a) On January 1, the company retired a piece of machinery that was purchased on January 1, 2009 for $6 000. It had a useful life of six years and no residual value.b) On June 30, the company sold a computer purchased on January 1, 2010. It was sold for $600. The computer cost $4 000 and had a useful life of six years with a residual value of $250.c) On January 1, the company discarded a delivery truck that was purchased on January 1, 2010. The truck cost $30 000. It was amortized based on a six-year useful life with a $3 000 residual value.Zaara's Mining Co.Zaara's Mining Co purchased a mine for $8 million on July 1, 2014. The mine will generate an estimated 30 million tonnes of nickel. The mine has a residual value of $500 000, and the restoration cost at the end of the mine's useful life is estimated at $600 000. In the first year, 5 million tonnes of nickel were extracted.Prepare the journal entry to record the amortization expense for the first year, ended June 30, 2015.Beaufort Corporation The following are the transactions concerning the intangible assets of Beaufort Corporation for 2015:Jan 1 - Paid $200 000 for the artistic development of a trademark. Also paid $50 000 to register the trademark and for the associated legal fees. The useful life of the trademark is indefinite.Feb 15 - Purchased a patent with an eight year useful life for $480 000.March 15 - Goodwill was purchased along with the assets of another company for $50 000. It has an indefinite useful life.April 1 - Research and development costs were incurred amounting to $550 000. There is no marketable product identified to these costs.Dec 31 - Any amortization on the previous intangible assets is calculated and recorded.a) Record the journal entries for the above transactions.b) Record the entries needed for amortization of these assets as at December 31, 2015.PART B Short Answer Answer all questions in full sentences. 1. What factors must a company consider when choosing an amortization method to use for a fixed asset? Be sure to refer to IFRS and the effect of the decision on the financial statements. 2. What is the Capital Cost Allowance and how does it affect the amortization of assets? List of sides in order from shortest to longest Practise the Skill 5a) Josie is going to a conference in 48 hoursHow many days is this? 17 yo f presents with prolonged, excessive menstrual bleeding ocuuring irrgularly withing the past 6 months What is the most likely diagnosis? Which is larger in A and B Perform the indicated operation.-3(-8)(-2) let a= 8 2 10 8 6 14 4 0 4 and w= 2 2 2 . determine if w is in col(a). is w in nul(a)? Al Jebra is ordering his new bicycle from Brenda's Bike Boutique. Al knows that he wants the Grand Touring model which has 14 choices for the color, and comes with or without a sidecar, with or without phone mount. How many unique versions of the Grand Touring bike are available? Why was her literary career so unusual? 1.1Which of the following pairs can be classified as vectors?A. Friction and massB. Mass and inertiaC. Inertia and weightD. Weight and friction What is the location of birth for Elizabeth Freeman P.E.S.T.L.E Analysis of Australian house prices set to drop further as interest rates surge.600 words I need a resume of The peloponneso war In addressing important social problems, such as educational failure, substance abuse, depression, and unemployment an empirically conscious social worker should be aware that: Which federal law sets the minimum standards for most voluntarily established pension and health plans in private industry to provide protection for individuals in these plans? When Americans or Europeans expect the return on ________ assets to be high relative to the return on ________ assets, there is a ________ demand for dollar assets, everything else held constant. euro; dollar; higher dollar; euro; constant euro; dollar; lower dollar; euro; lower P-please. Help me with question 12 why are drugs like sleeping pills described as the opposite of amphetamines? Value-at-Risk (VaR) analysis should be complemented by stress-testing because stress testing: Select one: O a. Provides a precise maximum loss, expressed in dollars O b. Summarize the expected loss over a target horizon within a minimum confidence interval Oc. Assesses the behavior of portfolio at a 99 percent confidence level d. Identifies losses that go beyond the normal losses measured by VaR what specific parts of the brain have been affected by the injury so as to cause the muscle weakness and language problems? what specific parts of the brain have been affected by the injury so as to cause the muscle weakness and language problems? the superior area of her right motor cortex. the superior area of her left motor cortex and broca's area. the inferolateral area of her left motor cortex and wernicke's area. the left midlateral motor cortex.